What Causes Oscillatory Behavior in Sinusoidal Functions?

Click For Summary
The discussion centers on the oscillatory behavior of a sinusoidal function represented as 2sin(π/3 t - π/6) + 6. The user graphed the function and identified equilibrium points but struggles to explain the oscillation around these points, particularly how initial conditions affect the behavior. They attempted to derive a formula for oscillation conditions but encountered contradictions in their calculations. Further analysis revealed that oscillatory behavior occurs at inflection points and midpoints between them, raising questions about what is oscillating and the relevance of initial conditions. The user seeks clarity on these concepts to better understand the dynamics of the sinusoidal function.
rmiller70015
Messages
110
Reaction score
1

Homework Statement



This isn't really part of my homework, my homework was to draw a pretty graph, but I am curious about some behavior.
I was given a picture of a sinusoidal function. I found it was ##2sin(\frac{\pi}{3}t-\frac{\pi}{6}) + 6##. Then I used trig identities to get ##\sqrt{3}sin(\frac{\pi}{3}t) - cos(\frac{\pi}{3}t) +6## and set it equal to ##t## to get ##\sqrt{3}sin(\frac{\pi}{3}t) - cos(\frac{\pi}{3}t) - t =-6##. I then plotted ##\sqrt{3}sin(\frac{\pi}{3}t) - cos(\frac{\pi}{3}t) - t = 0## and ##y = -6## and found the intercepts to get my equilibrium points of 4.392, 7, and 8.

This is where I am having issues. The web diagram's behavior around the first equilibrium is sensitive to initial conditions. Graphically I found that when t = 4, 5, or 6, the diagram will oscillate around the fixed point indefinitely. I'm having trouble explaining why this is.

Homework Equations

The Attempt at a Solution


I've tried starting off by saying the oscillation condition is ##t_n = t_{n+2}## and ##t_{n+1} = t_{n+3}##.

I think that the updating formula is ##t_{n+1} = (1-n)t_n + (1-n)t_n[2sin(\frac{\pi}{3}t - \frac{\pi}{6})+6]##

Then,
##t_{n+1} = (1-n)t_n[7+2sin(\frac{\pi}{3}t - \frac{\pi}{6})] = t_{n+3} = (1-(n+2))t_{n+2}[7+2sin(\frac{\pi}{3}t-\frac{\pi}{6})]##
Division gives:
##\frac{t_n}{t_{n+2}} = \frac{-(1+n)t_{n+2}[7+2sin(\frac{\pi}{3}t - \frac{\pi}{6})]}{(1-n)t_n[7 + 2sin(\frac{\pi}{3}t - \frac{\pi}{6})]}##
But this ends up telling me that 1 = -1, so, I'm not sure what to do from here or if this was the correct way to do things. I need to find some expression that allows the oscillation conditions to be met.

Edit: Upon closer inspection it appears that this oscillatory behavior occurs at the inflection points and half way between the inflection points.
 
Last edited:
Physics news on Phys.org
I can't understand. If you fix t=4, then what is "oscillating"? And what initial conditions are you referring to?
 
Question: A clock's minute hand has length 4 and its hour hand has length 3. What is the distance between the tips at the moment when it is increasing most rapidly?(Putnam Exam Question) Answer: Making assumption that both the hands moves at constant angular velocities, the answer is ## \sqrt{7} .## But don't you think this assumption is somewhat doubtful and wrong?

Similar threads

  • · Replies 1 ·
Replies
1
Views
2K
  • · Replies 1 ·
Replies
1
Views
2K
  • · Replies 11 ·
Replies
11
Views
2K
  • · Replies 14 ·
Replies
14
Views
2K
Replies
3
Views
2K
Replies
3
Views
2K
  • · Replies 1 ·
Replies
1
Views
1K
  • · Replies 7 ·
Replies
7
Views
3K
  • · Replies 5 ·
Replies
5
Views
2K
  • · Replies 3 ·
Replies
3
Views
2K